getaltheorie 2

Opgave - IMOSL 2000 vraag 15

Voor een natuurlijk getal $n$, zij $d(n)$ het aantal verschillende positieve delers van $n$. Vind alle natuurlijke getallen $n$ zodat $d(n)^3=4n$.

Oplossing

Duidelijk is $n = 2k^3$ voor een zekere $k\in\mathbb{N}_0$. Dan moet $d(2k^3) = 2k$.

Als $k = 2^m$, met $m\in\mathbb{N}$, dan moet $(3m+2) = d(2^{3m+1}) = d(2k^3) = 2k = 2^{m+1}$. Het is niet moeilijk aan te tonen via volledige inductie dat $2^{m+1} > 3m+2$ als $m\geq 3$. Bijgevolg moet $m\in\{0,1,2\}$.
Voor $m=0$ krijgen we $2 = (3m+2) = 2^{m+1} = 2$ :yes: Er geldt dan dat $k=1$, dus $n=2$.
Voor $m=1$ krijgen we $5 = (3m+2) = 2^{m+1} = 4$. :no:
Voor $m=2$ krijgen we $8 = (3m+2) = 2^{m+1} = 8$. :yes: Er geldt dan dat $k=2^2$, dus $n=2^7$.

Stel nu dus dat $$k = \prod_{i=1}^m p_i^{\alpha_i},$$waarbij $p_i$ allemaal verschillende priemgetallen zijn waarvoor geldt dat $p_i < p_j\ \Leftrightarrow\ i < j$, $p_1 = 2$ en $\alpha_i \geq 1$, $\forall i\geq 2$. ($p_1, p_2, ..., p_m$ zijn dus de $m$ priemdelers van $k$) We veronderstellen wlog dat $m\geq 2$. (Want als $m=1$, dan is enkel $p_1 = 2$ een priemdeler, en dan verwijs ik naar bovenstaande paragraaf.) Er geldt dan dat $$(3\alpha_1+2)\prod_{i=2}^m (3\alpha_i+1) = d(2k^3) = 2k = 2^{\alpha_1+1}\prod_{i=2}^m p_i^{\alpha_i}$$
Beschouw deze vergelijking modulo 3, dan is $2 \equiv LHS\bmod{3}$, dus $3\not|\ k$. Bijgevolg is $p_2 \geq 5$. Voor alle $i\in\{2,3,...,m\}$ geldt er dus dat $p_i \geq 5$, i.e. $p_i^{\alpha_i} \geq 5^{\alpha_i}$. Het is niet moeilijk aan te tonen dat $5^{\alpha_i} > 3\alpha_i+1$ via volledige inductie, als $\alpha_i\geq 1$, hetgeen geldt.

Dus als $\alpha_1 \geq 3$, dan zal, ten eerste, $3\alpha_1+2 < 2^{\alpha_1+1}$ en, ten tweede, $3\alpha_i+1 < p_i^{\alpha_i}$, voor alle $1 < i \leq m$. Vermenigvuldig al deze strikte ongelijkheden om te bekomen dat de gegeven gelijkheid $d(2k^3) = 2k$ dan niet kan gelden.

Er moet dus gelden dat $\alpha_1 < 3$, i.e. $\alpha_1 \in\{0,1,2\}$. Voor $\alpha_1 = 0$ en $\alpha_1 = 2$ krijgen we echter weer dat $$\prod_{i=2}^m (3\alpha_i+1) = \prod_{i=2}^m p_i^{\alpha_i},$$hetgeen onmogelijk is. (Zie hierboven: $p_i^{\alpha_i} \geq 5^{\alpha_i} \geq 3\alpha_i+1$.)

Dus $\alpha_1 = 1$. We krijgen dan $$5\prod_{i=2}^m (3\alpha_i+1) = 4\prod_{i=2}^m p_i^{\alpha_i}$$Modulo 5 om te bekomen dat $p_2 = 5$. Als $\alpha_2 \geq 2$, dan is $5(3\alpha_2+1) \leq 4\cdot 5^{\alpha_2}$, zodat we weer tot een contradictie komen. Bijgevolg moet $\alpha_2 = 1$. Dan krijgen we $$\prod_{i=3}^m (3\alpha_i+1) = \prod_{i=3}^m p_i^{\alpha_i},$$hetgeen enkel mogelijk is als beide producten eigenlijk niets voorstellen, i.e. $m<3$. Dit betekent dat $m=2$, dus is $k = 2^{\alpha_1}5^{\alpha_2} = 2\cdot 5$ en bijgevolg $n = 2^45^3$.